robowarren
Thanks Received: 0
Jackie Chiles
Jackie Chiles
 
Posts: 26
Joined: October 19th, 2011
 
 
 

Q9 - In some jurisdictions, lawmakers

by robowarren Thu Feb 09, 2012 3:16 pm

I had narrowed it down to A and D and in the end picked D. Looking at D, I see why it is wrong, because it says "most" and the stimulus says "some. BUT then, I dont see how B can be right. Can "in general" be equated with "some"?

Thanks you!
User avatar
 
ManhattanPrepLSAT1
Thanks Received: 1909
Atticus Finch
Atticus Finch
 
Posts: 2851
Joined: October 07th, 2009
 
 
 

Re: Q9 - In some jurisdictions, lawmakers

by ManhattanPrepLSAT1 Sat Feb 11, 2012 1:00 am

Good question, and thanks for bringing it to the forum! Actually, answer choice (D) is wrong for a different reason than the word "most". The issue with answer choice (D) is that it fails to connect the harm caused by the two acts with the penalty. Notice that answer choice (A) does this.

The argument tries concludes that some jurisdictions consider the harm from bribery and theft to be equal. Why? Because the penalty for both is equal. The argument assumes a relationship between the harm of a crime and the penalty that should be assessed for committing that crime. Answer choice (A) connects these ideas.

Let's look at the incorrect answers:

(B) connects the legality of an act with the harm resulting from an act. But legality is not the same as penalty.
(C) connects the harm of an act with the penalty for that act, but does so only for some extreme instances - not necessarily the ones discussed and only after the fact. The argument is about the standard penalty for bribery and theft.
(D) fails to connect the harm of each of these acts with the penalty.
(E) fails to suggest that the harm from bribery and theft is proportional to the penalty. So while we know the penalty is the same, we do not know the harm is the same.

Hope that helps!
 
donghai819
Thanks Received: 7
Elle Woods
Elle Woods
 
Posts: 65
Joined: September 25th, 2015
 
 
 

Re: Q9 - In some jurisdictions, lawmakers

by donghai819 Mon Jan 25, 2016 11:51 am

Unlike many weaken/strengthen questions that involved causation, this question asks us to link premise and conclusion. I do feel that this one is something that stands between principle and sufficient assumption questions.
 
erikwoodward10
Thanks Received: 9
Elle Woods
Elle Woods
 
Posts: 69
Joined: January 26th, 2014
 
 
 

Re: Q9 - In some jurisdictions, lawmakers

by erikwoodward10 Sat Jul 16, 2016 1:27 pm

This is a weird one. I see that the argument is missing the connection between harm and penalty, and understand that A makes this answer choice explicit, thus strengthening by making known a (necessary?) assumption of the argument. But on my BR D really caught my eye.

In reviewing this question I realized something about D. I initially thought that it could also strengthen (but not nearly as much as A) but supporting the premise that the harm of the two crimes is proportional. But upon a closer analysis, I realized that the stimulus talks about "harm resulting from theft" and D talks about harm to a "victim" of theft. These aren't the same thing, so even if they are analogous that doesn't really support the premise at all. The "harm resulting from theft" could include harm to society--crime rates go up, violence goes up, etc.--and D specifically talks about the harm to the "victim". Subtle shift that makes this wrong.